This preview shows page 1 out of 4 pages.

Save
View full document
View full document
Premium Document
Do you want full access? Go Premium and unlock all 4 pages.
Access to all documents
Download any document
Ad free experience
Premium Document
Do you want full access? Go Premium and unlock all 4 pages.
Access to all documents
Download any document
Ad free experience

Unformatted text preview:

STAT 418HW4 SolutionsSections 3.4-3.537, 39, 43, 50, 52, 55, 64, 65, 78, TE 1, TE 637. Let H = {Heads on first flip}, F = {Fair coin}, and T = {Two-headed coin}. Notethat F and T form a partition.a. By Bayes rule,P (F |H) =P (H|F )P (F )P (H|F )P (F ) + P (H|T )P (T )=.5 · .5.5 · .5 + 1 · .5=13b. Let HH = {Heads, Heads in 2 flips of the coin}. By independence, P (HH|F ) =14.P (F |HH) =P (HH|F )P (F )P (HH|F )P (F ) + P (HH|B)P (B)=.25 · .5.25 · .5 + 1 · .5=15c. Because the two-headed coin can never flip tails, P (F |HHT ) = 1.39. Let A = {Accident prone}, A1= {Accident in first year}, and A2= {Accident insecond year}. Find P (A2|¯A1).P (A2|¯A1) =P (A2∩¯A1)P (¯A1).By the law of total probability,P (A2|¯A1) =P (A2∩¯A1∩ A)P (¯A1)+P (A2∩¯A1∩¯A)P (¯A1)=P (A2∩¯A1∩ A)/P (¯A1∩ A)P (¯A1)/P (¯A1∩ A)+P (A2∩¯A1∩¯A)/P (¯A1∩¯A)P (¯A1)/P (¯A1∩¯A)= P (A2|¯A1∩ A)P (A|¯A1) + P (A2|¯A1∩¯A)P (¯A|¯A1).Given:P (A2|¯A1∩A) = .4, P (A2|¯A1∩¯A) = .2, P (A1|A) = .4, P (¯A1|¯A) = 1−.2 = .8, P (A) = .3, P (A1) = .26.Therefore,P (A|¯A1) =P (¯A1|A)P (A)P (¯A1)=(1 − .4) · .31 − .26= .243P (¯A|¯A1) =P (¯A1|¯A)P (¯A)P (¯A1)=.8 · (1 − .3)1 − .26= .757P (A2|¯A1) = .4 · .243 + .2 · .757 = .2486.43. Let H = {Heads flipped}, F = {Fair coin}, B = {Biased coin}, and T = {Two-headedcoin}. Note that F , B, and T form a partition. Given:P (H|F ) =12, P (F ) =13, P (H|B) =34, P (B) =13, P (H|T ) = 1, P (T ) =131By Bayes rule,P (T |H) =P (H|T )P (T )P (H|T )P (T ) + P (H|B)P (B) + P(H|F)P(F)=4950. Let G = {Good risks}, A = {Average risks}, B = {Bad risks}, and C = {Accident}.Note that G, A, and B form a partition. Given:P (C|G) = .05, P (C|A) = .15, P (C|B) = .3, P (G) = .2, P (A) = .5, P (B) = .3.Find:P (C), P (G|¯C), P (A|¯C)By the law of total probability,P (C) = P (C|G)P(G) + P(C|A)P (A) + P (C|B)P(B) = .175.P (G|¯C) =P (¯C|G)P (G)1 − P (C)=(1 − .05) · .21 − .175= .23P (A|¯C) =P (¯C|A)P (A)1 − P (C)=(1 − .15) · .51 − .175= .5252. Let M, T , W, R, and F be the events that mail is received on that day. Let A ={Accepted} and J = {Rejected}. Note that A and J form a partition.a. By the law of total probability,P (M) = P (M|A)P (A) + P (M|J)P (J) = .15 · .6 + .05 · .4 = .11b.P (T |¯M) =P (T ∩¯M)P (¯M)=P (T )1 − P (M)=P (T |A)P (A) + P (T |J)P (J)1 − P (M)=.2 · .6 + .1 · .41 − .11= .180c. By Bayes rule,P (A|¯M ∩¯T ∩¯W ) =P (¯M ∩¯T ∩¯W |A)P (A)P (¯M ∩¯T ∩¯W |A)P (A) + P (¯M ∩¯T ∩¯W |J)P (J)=(1 − .15 − .2 − .25) · .6(1 − .15 − .2 − .25) · .6 + (1 − .05 − .1 − .1) · .4= .444.d. By Bayes rule,P (A|R) =P (R|A)P (A)P (R|A)P (A) + P (R|J)P (J)=.15 · .6.15 · .6 + .15 · .4= .6e. By Bayes rule,P (A|¯M ∩¯T ∩¯W ∩¯R ∩¯F ) =P (¯M ∩¯T ∩¯W ∩¯R ∩¯F |A)P (A)P (¯M ∩¯T ∩¯W ∩¯R ∩¯F |A)P (A) + P (¯M ∩¯T ∩¯W ∩¯R ∩¯F |J)P (J)=(1 − .15 − .2 − .25 − .15 − .1) · .6(1 − .15 − .2 − .25 − .15 − .1) · .6 + (1 − .05 − .1 − .1 − .15 − .2) · .4= .36255. Let x = the total number of sophomore girls. Let G = {Girl selected} and S ={Sophomore selected}. By the defintion of independence, sex and class, or equivalently, Gand S, are independent iff P (G|S) = P (G). Thereforex6 + x=6 + x16 + x16x + x2= 36 + 12x + x24x = 36x = 9.64. Let C = {Correct answer}, W = {Wife answers}, Wc= {Wife correct}, and Hc={Husband correct}. W and¯W form a partition of the entire sample space. Hc∩ Wc,¯Hc∩ Wc,Hc∩¯Wc, and¯Hc∩¯Wcalso form a partition of the sample space; when they are both corrector incorrect, they agree.Under situation (a), assuming choice is at random,P (C) = P (C|W )P (W ) + P (C|¯W )P (¯W ) = p ·12+ p ·12= p.Under situation (b),P (C) = P (C|Hc∩ Wc)P (Hc∩ Wc) + P (C|¯Hc∩ Wc)P (¯Hc∩ Wc) + P (C|Hc∩¯Wc)P (Hc∩¯Wc)+ P (C|¯Hc∩¯Wc)P (¯Hc∩¯Wc)= 1 · p · p +12· (1 − p) · p +12· p · (1 − p) + 0 · (1 − p) · (1 − p)= p2+ p − p2= p.Both strategies give the same probability of winning.65. Let A = {they agree}.a.P (C|A) =P (C ∩ A)P (A)=P (Hc∩ Wc)P (Hc∩ Wc) + P (¯Hc∩¯Wc)=p2p2+ (1 − p)2= .692b.P (C|¯A) =C ∩¯A)P (¯A)=P (C ∩ [(Hc∩¯Wc) ∪ (¯Hc∩ Wc)])1 − P (A)=P (C ∩ Hc∩¯Wc) + P (C ∩¯Hc∩ Wc)1 − P (A)=12· p · (1 − p) +12· (1 − p) · p1 − (2p2− 2p + 1)=p(1 − p)2p(1 − p)=12.378.a. Let F = {4 games are played}, A4= {A wins in 4 games}, and B4= {B wins in 4games}.P (F ) = P (A4) + P (B4)= p(1 − p)p2+ (1 − p)p3+ (1 − p)p(1 − p)2+ p(1 − p)3= 2p(1 − p)[p2+ (1 − p)2].b. Partition the sample space based on the outcome of the first two games. Let A = {Awins}, aa = {A wins first 2 games}, ab = {A wins first, B wins second}, ba = {B winsfirst, A wins second}, and bb = {B wins first 2 games}. By the law of total probability,P (A) = P (A|aa)P (aa) + P (A|ab)P (ab) + P (A|ba)P (ba) + P (A|bb)P (bb).If either ab or ba occur, the players are even and because the games are independent,this is equivalent to starting over. Therefore P (A) = P (A|ab) = P (A|ba). This yieldsP (A) = 1 · p2+ 2P (A)p(1 − p) + 0 · (1 − p)2P (A) − 2P (A)p(1 − p) = p2P (A) =p21 − 2p + 2p2.TE 1. Suppose P (A) > 0,P (A ∩ B|A ∪ B) =P (A ∩ B ∩ (A ∪ B))P (A ∪ B)=P ((A ∩ B) ∪ (A ∩ B)P (A ∪ B)=P (A ∩ B)P (A ∪ B).Because P (A ∪ B) ≥ P (A) and P (A) > 0,P (A ∩ B)P (A ∪ B)≤P (A ∩ B)P (A)=P (A ∩ B ∩ A)P (A)= P (A ∩ B|A)TE 6. By DeMorgan’s laws,P (∪ni=1Ei) = P (∩ni=1¯Ei) = 1 − P (∩ni=1¯Ei) = 1 −nYi=1(1 − P


View Full Document

PSU STAT 418 - hw4

Documents in this Course
Load more
Download hw4
Our administrator received your request to download this document. We will send you the file to your email shortly.
Loading Unlocking...
Login

Join to view hw4 and access 3M+ class-specific study document.

or
We will never post anything without your permission.
Don't have an account?
Sign Up

Join to view hw4 2 2 and access 3M+ class-specific study document.

or

By creating an account you agree to our Privacy Policy and Terms Of Use

Already a member?